Está en la página 1de 8

Desigualdades

cuadráticas de una
variable
MATEMÁTICA APLICADAS A LA ADMINISTRACIÓN Y A
LA ECONOMÍA

Dayana Chacon Parra


Código: 156204803
Lizeth Katerine Titimbo Lemus
Código: 156204870
38. (Inversiones) Un accionista invierte $100
(dólares) a un interés anual del R por ciento y otros
$100 (dólares) al 2R porciento anual. Si el valor de
las dos inversiones debe ser al menos $224.80
después de dos años, ¿qué restricciones deben
establecerse sobre R?
𝐼1 = 𝐼𝑣𝑒𝑟𝑠𝑖ó𝑛 1 𝐼2 = 𝐼𝑛𝑣𝑒𝑟𝑖𝑠ó𝑛 2 𝐼𝑛𝑡𝑒𝑟𝑒𝑠 1 = 𝑅%

𝐼1 = $100 𝐼2 = $100 𝐼𝑛𝑡𝑒𝑟𝑒𝑠 2 = 2𝑅%

𝐴ñ𝑜 1
𝐺𝑎𝑛𝑎𝑛𝑐𝑖𝑎 𝐼1 = 𝐼𝑣𝑒𝑟𝑠𝑖ó𝑛 1 + 𝐼𝑛𝑡𝑒𝑟𝑒𝑠 𝐺𝑎𝑛𝑎𝑛𝑐𝑖𝑎 𝐼2 = 𝐼𝑣𝑒𝑟𝑠𝑖ó𝑛 2 + 𝐼𝑛𝑡𝑒𝑟𝑒𝑠
𝑅 2𝑅
𝐺𝑎𝑛𝑎𝑛𝑐𝑖𝑎 𝐼1 = 𝐼1 + 𝐼1 𝐺𝑎𝑛𝑎𝑛𝑐𝑖𝑎 𝐼2 = 𝐼2 + 𝐼2
100 100
𝑅 2𝑅
𝐺𝑎𝑛𝑎𝑛𝑐𝑖𝑎 𝐼1 = 100 + 100 𝐺𝑎𝑛𝑎𝑛𝑐𝑖𝑎 𝐼2 = 100 + 100
100 100
𝐺𝑎𝑛𝑎𝑛𝑐𝑖𝑎 𝐼1 = 100 + 𝑅 𝐺𝑎𝑛𝑎𝑛𝑐𝑖𝑎 𝐼2 = 100 + 2𝑅

𝐺𝑎𝑛𝑎𝑛𝑐𝑖𝑎 𝑡𝑜𝑡𝑎𝑙 𝑎ñ𝑜 1 = 𝐺𝑎𝑛𝑎𝑛𝑐𝑖𝑎 1 + 𝐺𝑎𝑛𝑎𝑛𝑐𝑖𝑎 2


NOTA:
𝐺𝑎𝑛𝑎𝑛𝑐𝑖𝑎 𝑡𝑜𝑡𝑎𝑙 𝑎ñ𝑜 1 = (100 + 𝑅) + (100 +2R)
𝐺𝑎𝑛𝑎𝑛𝑐𝑖𝑎 𝑡𝑜𝑡𝑎𝑙 𝑎ñ𝑜 1 = 200 + 3R
𝐴ñ𝑜 2

𝐺𝑎𝑛𝑎𝑛𝑐𝑖𝑎 𝑡𝑜𝑡𝑎𝑙 𝐼1 = 𝐺𝑎𝑛𝑎𝑛𝑐𝑖𝑎 𝐼1 + 𝐼𝑛𝑡𝑒𝑟𝑒𝑠


𝑅
𝐺𝑎𝑛𝑎𝑛𝑐𝑖𝑎 𝑡𝑜𝑡𝑎𝑙 𝐼1 = 𝐺𝑎𝑛𝑎𝑛𝑐𝑖𝑎 𝐼1 + 𝐺𝑎𝑛𝑎𝑛𝑐𝑖𝑎 𝐼1
100
𝑅
𝐺𝑎𝑛𝑎𝑛𝑐𝑖𝑎 𝑡𝑜𝑡𝑎𝑙 𝐼1 = (100 + 𝑅) + (100 + 𝑅)
100
𝑅 𝑅2
𝐺𝑎𝑛𝑎𝑛𝑐𝑖𝑎 𝑡𝑜𝑡𝑎𝑙 𝐼1 = 100 + 𝑅 + 100 +
100 100
𝑅2
𝐺𝑎𝑛𝑎𝑛𝑐𝑖𝑎 𝑡𝑜𝑡𝑎𝑙 𝐼1 = 100 + 𝑅 + 𝑅 +
100
𝑅2
𝐺𝑎𝑛𝑎𝑛𝑐𝑖𝑎 𝑡𝑜𝑡𝑎𝑙 𝐼1 = 100 + 2𝑅 +
100
𝐺𝑎𝑛𝑎𝑛𝑐𝑖𝑎 𝑡𝑜𝑡𝑎𝑙 𝐼2 = 𝐺𝑎𝑛𝑎𝑛𝑐𝑖𝑎 𝐼2 + 𝐼𝑛𝑡𝑒𝑟𝑒𝑠
2𝑅
𝐺𝑎𝑛𝑎𝑛𝑐𝑖𝑎 𝑡𝑜𝑡𝑎𝑙 𝐼2 = 𝐺𝑎𝑛𝑎𝑛𝑐𝑖𝑎 𝐼2 + 𝐺𝑎𝑛𝑎𝑛𝑐𝑖𝑎 2
100
2𝑅
𝐺𝑎𝑛𝑎𝑛𝑐𝑖𝑎 𝑡𝑜𝑡𝑎𝑙 𝐼2 = (100 + 2𝑅) + (100 + 2𝑅)
100
2𝑅 4𝑅2
𝐺𝑎𝑛𝑎𝑛𝑐𝑖𝑎 𝑡𝑜𝑡𝑎𝑙 𝐼2 = 100 + 2𝑅 + 100 +
100 100
4𝑅2
𝐺𝑎𝑛𝑎𝑛𝑐𝑖𝑎 𝑡𝑜𝑡𝑎𝑙 𝐼2 = 100 + 2𝑅 + 2𝑅 +
100
4𝑅2
𝐺𝑎𝑛𝑎𝑛𝑐𝑖𝑎 𝑡𝑜𝑡𝑎𝑙 𝐼2 = 100 + 4𝑅 +
100
𝐺𝑎𝑛𝑎𝑛𝑐𝑖𝑎 𝑡𝑜𝑡𝑎𝑙 𝑎ñ𝑜 2 = 𝐺𝑎𝑛𝑎𝑛𝑐𝑖𝑎 𝑡𝑜𝑡𝑎𝑙 𝐼1 + 𝐺𝑎𝑛𝑎𝑛𝑐𝑖𝑎 𝑡𝑜𝑡𝑎𝑙 𝐼2
𝑅2 4𝑅2
𝐺𝑎𝑛𝑎𝑛𝑐𝑖𝑎 𝑡𝑜𝑡𝑎𝑙 𝑎ñ𝑜 2 = (100 + 2𝑅 + )+ (100 + 4𝑅 + )
100 100
5𝑅2
𝐺𝑎𝑛𝑎𝑛𝑐𝑖𝑎 𝑡𝑜𝑡𝑎𝑙 𝑎ñ𝑜 2 = + 6𝑅 + 200
100

Pero conocemos que el valor de estas dos inversiones después de dos años
debe ser al menos $224.80. Entonces,

5𝑅2
+ 6𝑅 + 200 ≥ 224.80
100
5𝑅2
+ 6𝑅 + 200 − 224.80 ≥ 224.80 − 224.80
100
5𝑅2
100 + 6𝑅 − 24.80 ≥ 100 ∗ 0
100
5𝑅2
100 + 100 6𝑅 + 100(24.80) ≥ 100 ∗ 0
100
5𝑅2 + 600𝑅 − 2480 ≥ 0
5𝑅2 + 600𝑅 − 2480 ≥ 0
5𝑅2 + 600𝑅 − 2480 = 0

Hallamos los valores de R por medio de la Formula Cuadrática,

−600 ± 6002 − 4 5 −2480


R=
2 5
−600 ± 360000 − −49600
R=
10
−600 ± 409600
R=
10
−600 + 409600
R=
10
R=4
−600 − 409600
R=
10
𝑅 = −124
R=4 𝑅 = −124

VERDADERO FALSO VERDADERO

-∞ -124 0 4 ∞

Vamos a tomar un punto de cada intervalo y a probarlo en la desigualdad (5𝑅2 +

Si R= -200
5(−200)2 +600(−200) − 2480 ≥ 0 RESPUESTA:
77520 ≥ 0
(−∞, 124] 𝑈 [4, ∞)
Si R= 0
5(0)2 +600(0) − 2480 ≥ 0 Pero como estamos hablando de una
−2480 ≥ 0
tasa de interés solo tomamos el
Si R= 5 intervalo de [4, ∞)
5(5)2 +600(5) − 2480 ≥ 0 Así tenemos que para que el valor de
645 ≥ 0 estas dos inversiones después de dos
años sea de al menos $224.80 la tasa de
interés debe ser igual o mayor al 4%

También podría gustarte